2
$\begingroup$

I apologize if this question is meaningless or trivial:

What are examples of Algebras admitting quantifier elimination? Especially are there Groups admitting quantifier elimination?

I need to say some words about my motivation: last week I was proved some results concerning relatively free algebras in varieties, as an example the following result was proved:

Theorem (false) Let $\mathbf{V}$ be a variety of algebras and $X$ be a set. Then the relatively free algebra $F_{\mathbf{V}}(X)$ is $u_{\omega}$-compact.

A communication with Anton Klaychko revealed to me that my argument had mistake. The mistake was funny: I was replaced $\forall x \varphi(x) \Leftrightarrow \forall x \psi(x)$ by $\forall x( \varphi(x)\Leftrightarrow \psi(x))$ implicitly in my proof. Today, I learned some elementary facts about quantifier elimination and assuming that $\mathbf{V}=Var(A)$ with $A$ an algebra admitting quantifier elimination, I modified my proof. Now I have a correct proof of the above theorem with the extra assumption:

1- $\mathbf{V}=Var(A)$,

2- $A$ admits quantifier elimination.

I need some examples of such algebras to complete my work. Before I learned about some well-known examples such as algebraically closed fields and some Boolean algebras (and some other examples which are not algebras and so are not relevant).

$\endgroup$
3
  • 3
    $\begingroup$ Any structure can be encoded as an algebra while preserving quantifier elimination. Just fix two constants and replace all predicates with their characteristic functions. $\endgroup$ Jan 25, 2014 at 22:45
  • $\begingroup$ It's worth noting that in general, quantifier elimination and completeness go hand in hand... $\endgroup$
    – cody
    Jul 7, 2015 at 1:02
  • $\begingroup$ And decidability of course. $\endgroup$
    – cody
    Jul 7, 2015 at 1:07

1 Answer 1

6
$\begingroup$

The finite groups with quantifier elimination are classified in

Cherlin, Gregory; Felgner, Ulrich, Homogeneous finite groups. J. London Math. Soc. (2) 62 (2000), no. 3, 784–794.

$\endgroup$

Your Answer

By clicking “Post Your Answer”, you agree to our terms of service and acknowledge you have read our privacy policy.

Not the answer you're looking for? Browse other questions tagged or ask your own question.